Is Banach fixed point theorem a necessary and sufficient condition for the existence of a fixed point











up vote
1
down vote

favorite












Banach fixed point theorem requires a contraction mapping from a metric space into itself, but when I was learning some machine learning algorithms, some questions rise above:
k-means is an algorithm for clustering, it can be proved that this method will converge, but the proof of convergence of the algorithm doesn't involve fixed point theorem. I feel the iteration of the centering point for each cluster in this method is very similar the iteration of the fixed point iteration steps so I tried to prove this convergence using Banach fixed point theorem. However I couldn't construct a contraction mapping in this problem. So I guess if the iteration steps in k-means is not a contraction at all. In order to test this assumption, I generate some random number on my computer and use the k-means steps to cluster and calculate the norm distance between each iteration point.To my surprise, it is NOT a contraction!but it does converge in finite steps. I think the convergence shows the existence of the fixed point under this iteration. So with these facts, can I say the Bananch Fixed point theorem gives a sufficient condition on the existence of a fixed point, but it might not be necessary?










share|cite|improve this question






















  • If you're asking if contraction in each step is necessary for convergence, then yes you're right - it is not necessary
    – OnceUponACrinoid
    Nov 17 at 7:24










  • $mathbb R$ is a Banach space and $f(x)=2x$ is map with a unique fixed point. It is not a contraction.
    – Kavi Rama Murthy
    Nov 17 at 12:09















up vote
1
down vote

favorite












Banach fixed point theorem requires a contraction mapping from a metric space into itself, but when I was learning some machine learning algorithms, some questions rise above:
k-means is an algorithm for clustering, it can be proved that this method will converge, but the proof of convergence of the algorithm doesn't involve fixed point theorem. I feel the iteration of the centering point for each cluster in this method is very similar the iteration of the fixed point iteration steps so I tried to prove this convergence using Banach fixed point theorem. However I couldn't construct a contraction mapping in this problem. So I guess if the iteration steps in k-means is not a contraction at all. In order to test this assumption, I generate some random number on my computer and use the k-means steps to cluster and calculate the norm distance between each iteration point.To my surprise, it is NOT a contraction!but it does converge in finite steps. I think the convergence shows the existence of the fixed point under this iteration. So with these facts, can I say the Bananch Fixed point theorem gives a sufficient condition on the existence of a fixed point, but it might not be necessary?










share|cite|improve this question






















  • If you're asking if contraction in each step is necessary for convergence, then yes you're right - it is not necessary
    – OnceUponACrinoid
    Nov 17 at 7:24










  • $mathbb R$ is a Banach space and $f(x)=2x$ is map with a unique fixed point. It is not a contraction.
    – Kavi Rama Murthy
    Nov 17 at 12:09













up vote
1
down vote

favorite









up vote
1
down vote

favorite











Banach fixed point theorem requires a contraction mapping from a metric space into itself, but when I was learning some machine learning algorithms, some questions rise above:
k-means is an algorithm for clustering, it can be proved that this method will converge, but the proof of convergence of the algorithm doesn't involve fixed point theorem. I feel the iteration of the centering point for each cluster in this method is very similar the iteration of the fixed point iteration steps so I tried to prove this convergence using Banach fixed point theorem. However I couldn't construct a contraction mapping in this problem. So I guess if the iteration steps in k-means is not a contraction at all. In order to test this assumption, I generate some random number on my computer and use the k-means steps to cluster and calculate the norm distance between each iteration point.To my surprise, it is NOT a contraction!but it does converge in finite steps. I think the convergence shows the existence of the fixed point under this iteration. So with these facts, can I say the Bananch Fixed point theorem gives a sufficient condition on the existence of a fixed point, but it might not be necessary?










share|cite|improve this question













Banach fixed point theorem requires a contraction mapping from a metric space into itself, but when I was learning some machine learning algorithms, some questions rise above:
k-means is an algorithm for clustering, it can be proved that this method will converge, but the proof of convergence of the algorithm doesn't involve fixed point theorem. I feel the iteration of the centering point for each cluster in this method is very similar the iteration of the fixed point iteration steps so I tried to prove this convergence using Banach fixed point theorem. However I couldn't construct a contraction mapping in this problem. So I guess if the iteration steps in k-means is not a contraction at all. In order to test this assumption, I generate some random number on my computer and use the k-means steps to cluster and calculate the norm distance between each iteration point.To my surprise, it is NOT a contraction!but it does converge in finite steps. I think the convergence shows the existence of the fixed point under this iteration. So with these facts, can I say the Bananch Fixed point theorem gives a sufficient condition on the existence of a fixed point, but it might not be necessary?







functional-analysis banach-spaces machine-learning fixed-point-theorems






share|cite|improve this question













share|cite|improve this question











share|cite|improve this question




share|cite|improve this question










asked Nov 17 at 7:21









Nancy Zhang

255




255












  • If you're asking if contraction in each step is necessary for convergence, then yes you're right - it is not necessary
    – OnceUponACrinoid
    Nov 17 at 7:24










  • $mathbb R$ is a Banach space and $f(x)=2x$ is map with a unique fixed point. It is not a contraction.
    – Kavi Rama Murthy
    Nov 17 at 12:09


















  • If you're asking if contraction in each step is necessary for convergence, then yes you're right - it is not necessary
    – OnceUponACrinoid
    Nov 17 at 7:24










  • $mathbb R$ is a Banach space and $f(x)=2x$ is map with a unique fixed point. It is not a contraction.
    – Kavi Rama Murthy
    Nov 17 at 12:09
















If you're asking if contraction in each step is necessary for convergence, then yes you're right - it is not necessary
– OnceUponACrinoid
Nov 17 at 7:24




If you're asking if contraction in each step is necessary for convergence, then yes you're right - it is not necessary
– OnceUponACrinoid
Nov 17 at 7:24












$mathbb R$ is a Banach space and $f(x)=2x$ is map with a unique fixed point. It is not a contraction.
– Kavi Rama Murthy
Nov 17 at 12:09




$mathbb R$ is a Banach space and $f(x)=2x$ is map with a unique fixed point. It is not a contraction.
– Kavi Rama Murthy
Nov 17 at 12:09















active

oldest

votes











Your Answer





StackExchange.ifUsing("editor", function () {
return StackExchange.using("mathjaxEditing", function () {
StackExchange.MarkdownEditor.creationCallbacks.add(function (editor, postfix) {
StackExchange.mathjaxEditing.prepareWmdForMathJax(editor, postfix, [["$", "$"], ["\\(","\\)"]]);
});
});
}, "mathjax-editing");

StackExchange.ready(function() {
var channelOptions = {
tags: "".split(" "),
id: "69"
};
initTagRenderer("".split(" "), "".split(" "), channelOptions);

StackExchange.using("externalEditor", function() {
// Have to fire editor after snippets, if snippets enabled
if (StackExchange.settings.snippets.snippetsEnabled) {
StackExchange.using("snippets", function() {
createEditor();
});
}
else {
createEditor();
}
});

function createEditor() {
StackExchange.prepareEditor({
heartbeatType: 'answer',
convertImagesToLinks: true,
noModals: true,
showLowRepImageUploadWarning: true,
reputationToPostImages: 10,
bindNavPrevention: true,
postfix: "",
imageUploader: {
brandingHtml: "Powered by u003ca class="icon-imgur-white" href="https://imgur.com/"u003eu003c/au003e",
contentPolicyHtml: "User contributions licensed under u003ca href="https://creativecommons.org/licenses/by-sa/3.0/"u003ecc by-sa 3.0 with attribution requiredu003c/au003e u003ca href="https://stackoverflow.com/legal/content-policy"u003e(content policy)u003c/au003e",
allowUrls: true
},
noCode: true, onDemand: true,
discardSelector: ".discard-answer"
,immediatelyShowMarkdownHelp:true
});


}
});














draft saved

draft discarded


















StackExchange.ready(
function () {
StackExchange.openid.initPostLogin('.new-post-login', 'https%3a%2f%2fmath.stackexchange.com%2fquestions%2f3002058%2fis-banach-fixed-point-theorem-a-necessary-and-sufficient-condition-for-the-exist%23new-answer', 'question_page');
}
);

Post as a guest















Required, but never shown






























active

oldest

votes













active

oldest

votes









active

oldest

votes






active

oldest

votes
















draft saved

draft discarded




















































Thanks for contributing an answer to Mathematics Stack Exchange!


  • Please be sure to answer the question. Provide details and share your research!

But avoid



  • Asking for help, clarification, or responding to other answers.

  • Making statements based on opinion; back them up with references or personal experience.


Use MathJax to format equations. MathJax reference.


To learn more, see our tips on writing great answers.





Some of your past answers have not been well-received, and you're in danger of being blocked from answering.


Please pay close attention to the following guidance:


  • Please be sure to answer the question. Provide details and share your research!

But avoid



  • Asking for help, clarification, or responding to other answers.

  • Making statements based on opinion; back them up with references or personal experience.


To learn more, see our tips on writing great answers.




draft saved


draft discarded














StackExchange.ready(
function () {
StackExchange.openid.initPostLogin('.new-post-login', 'https%3a%2f%2fmath.stackexchange.com%2fquestions%2f3002058%2fis-banach-fixed-point-theorem-a-necessary-and-sufficient-condition-for-the-exist%23new-answer', 'question_page');
}
);

Post as a guest















Required, but never shown





















































Required, but never shown














Required, but never shown












Required, but never shown







Required, but never shown

































Required, but never shown














Required, but never shown












Required, but never shown







Required, but never shown







Popular posts from this blog

Biblatex bibliography style without URLs when DOI exists (in Overleaf with Zotero bibliography)

ComboBox Display Member on multiple fields

Is it possible to collect Nectar points via Trainline?